Data Interpretation - Online Test

Q1. Direction: In the following number series only one number is wrong. Find out the wrong number.
0, 7, 28, 63, 124, 215
Answer : Option D
Explaination / Solution:

The pattern is = 13 - 1, 23 - 1, 33 - 1, 43 - 1, 53 - 1, 63 - 1 
So the wrong term 28 is replace by = 33 - 1 = 27

Q2. Direction: What will come in place of the question mark (?) in the following number series?
32, 49, 83, 151, 287, 559, ?
Answer : Option C
Explaination / Solution:

32 + 1*17 = 32 + 17 = 49; 
49 + 2*17 = 49 + 34 = 83 
83 + 4*17 = 83 + 68 = 151; 
151 + 8*17 = 151 + 136 = 287 
287 + 16*17 = 287 + 272 = 559; 
559 + 32*17 = 559 + 544 = 1103

Q3. Direction: What will come in place of the question mark (?) in the following number series?
1050, 420, 168, 67.2, 26.88, 10.752  ?
Answer : Option A
Explaination / Solution:

1050 *2/5 = 420; 
420 *2/5 = 168 
168 *2/5 = 67.2
67.2 *2/5 = 26.88
26.88 *2/5 = 10.752 
10.752 *2/5 = 4.3008

Q4. Direction: What will come in place of the question mark (?) in the following number series?
45030     9000    1795    355    68    ?
Answer : Option A
Explaination / Solution:

(45030/5) - 6 = 9000
(9000/5) - 5 = 1795 
(1795/5) - 4 = 355
(355/5) - 3 = 68 
(68/5) - 2 = 11.6

Q5. Direction: What should come in place of the question mark (?) in the following number series?
17 21 37 73 137 (?)
Answer : Option D
Explaination / Solution:

The pattern of the Sequence & Series is: 
17 + 22 = 17 + 4 = 21 
21 + 42 = 21 +16 = 37 
37 + 62 = 37 + 36 = 73 
73 + 82 = 73 + 64 = 137 
137 + 102 = 137 + 100 = 237 

So, ? = 137 + 102 = 137 + 100 = 237

Q6. Direction: What will come in place of the question mark (?) in the following number series?
17, 22, 32, 47, 67, 92, ?
Answer : Option D
Explaination / Solution:

The given series is 
+ (5 × 1), 
+ (5 × 2), 
+ (5 × 3), 
+ (5 × 4), 
+ (5 × 5), 
+ (5 × 6)

Q7. Direction: What will come in place of the question mark (?) in the following number series?
311 300 278 245 201 146 ?
Answer : Option C
Explaination / Solution:

The given series is – (11 × 1), – (11 × 2) – (11 × 3), - (11 × 4), – (11 × 5), – (11 × 6)

Q8. Direction: What will come in place of the question mark (?) in the following number series?
 4, 9, 29, ?, 599, 3599
Answer : Option E
Explaination / Solution:

9 = 4×2+1= 8+1 
29 = 9×3+2 = 27+2 
? = 29×4 + 3 = 116+3 = 119 
599 = 119×5 + 4 = 595+4 
3599 = 599×6+5 = 3594+ 5

Q9. Direction: What will come in place of the question mark (?) in the following number series?
177 170 159 146 ? 110
Answer : Option C
Explaination / Solution:

The numbers given here have difference of prime numbers in accending order starting from 7.

177 – 170 = 7 
170 – 159 = 11
159 – 146 = 13 
Hence next number become 
146 – 17 = 129 
129 – 110 = 19 

Q10. Direction: In the following number series only one number is wrong. Find out the wrong number.
5, 6, 12, 40, 161, 806
Answer : Option C
Explaination / Solution:

The pattern is 5*1 + 1 = 6, 6*2 + 1 = 13, 13*3 + 1 = 40, 40*4 + 1 = 161, 161*5 + 1 = 806
So the wrong term 12 is replace by = 6*2 + 1 = 13